Clara lends half her collection of formal attires to her sister, Susan. Clara then buys four more attires. If she has 12 attires now, how many attires did Clara initially?

Answers

Answer 1

Answer:

16

Step-by-step explanation:

Number of attires Clara has initially = x

lends half her collection of formal attires to her sister

Attires remaining after lending = Number of attires Clara has initially - attires she lent her sister

= x - 1/2x

= 1/2x

Clara then buys four more attires

= 1/2x + 4

If she has 12 attires now

12 = 1/2x + 4

12 - 4 = 1/2x

8 = 1/2x

x = 8 ÷ 1/2

= 8 × 2/1

= 16

x = 16

Number of attires Clara has initially = x = 16


Related Questions

Mr. Austin decided to make a healthy snack for the 20 students in his class. He gave each student a dish of yogurt and divided 6 cups of strawberries equally among the dishes.
How many cups of strawberries did each student get in their yogurt?
Write your answer as a proper fraction or mixed number.

Answers

Answer:

3/10 of a cup

Step-by-step explanation:

Find how many cups of strawberries each student got by dividing 6 by 20:

= 6/20

Simplify this by dividing each number by 2:

= 3/10

So, each student got 3/10 of a cup of strawberries

(c) 0.34 L = _________________ ml ​

Answers

Answer:

340 ml

Step-by-step explanation:

The question is in the photo

Answers

Answer:

Step-by-step explanation:

This is a right triangle because angle V is 90. Draw out a right triangle and put V at the 90 degree angle. It doesn't matter where you put U or T; you get the same cos value for T regardless of how you place your other 2 angles. The things you need to know are that UT is the hypotenuse of the triangle, side VU is across from angle T, and side TV is across from angle U.

The cos ratio is side adjacent to the reference angle over the hypotenuse.

Our reference angle is T, so we are looking for the side next to T that is NOT the hypotenuse. This side measures 33; the hypotenuse measures 65, so the tangent ratio of T is

[tex]tanT=\frac{33}{65}[/tex]

If f(x)=3x^(2)+1 and G(x)=2x-3 what would f(f(x))

Answers

Answer:

f(f(x)) = 27[tex]x^{4}[/tex] + 18x² + 4

Step-by-step explanation:

To find f(f(x)) substitute x = f(x) into f(x) , that is

f(3x² + 1)

= 3(3x² + 1)² + 1 ← expand parenthesis using FOIL

= 3(9[tex]x^{4}[/tex] + 6x² + 1) + 1 ← distribute parenthesis by 3

= 27[tex]x^{4}[/tex] + 18x² + 3 + 1 ← collect like terms

= 27[tex]x^{4}[/tex] + 18x² + 4

Hello,

[tex](fof)(x)=f(f(x))\\\\=3(3x^2+1)^2+1\\\\=3(9x^4+6x^2+1)+1\\\\\boxed{=27x^4+18x^2+4}[/tex]

nth term of an AP is given by an expression 7n-2,find the common difference of this sequence if fisrt term of sequence is 1? a.2 b.-2 c-7 d7​

Answers

Step-by-step explanation:

there is something wrong with what your write here as problem description.

either there is something missing in the main expression, or in the answer options.

given your nth-term expression 7n-2, none of the 4 answer options can fit.

so, I thought, maybe there was a typo.

the possible interpretations of the expression could be

7n - 2

7(n-2)

[tex] {7}^{n} - 2[/tex]

[tex] {7}^{n - 2} [/tex]

we know that

a1 = 1

a2 = either

a1 + 2 = 3

a1 - 2 = -1

a1 + 7 = 8

a1 - 7 = -6

so, we can eliminate both negative answer options, because they would cause only negative sequence elements, but the main expression (neither of the 4 possibilities) does not allow that.

but also none of the 4 possibilities of the expression delivers any of the 4 values for a2 as described above.

7×2 - 2 = 14 - 2 = 12

7(2-2) = 7×0 = 0

7² - 2 = 49 - 2 = 47

[tex] {7}^{2 - 2} = {7}^{0} = 1[/tex]

similar not for a3.

so, if I consider your answer options right, we have 4 possible arithmetic sequences:

1. 1, 3, 5, 7, 9, 11, 13, 15, ...

2. 1, -1, -3, -5, -7, -9, -11, -13, ...

3. 1, 8, 15, 22, 29, 36, 43, 50, ...

4. 1, -6, -13, -20, -27, -34, -41, ...

the nth term expression is for

a. an = 2n - 1

b. an = 2×(-n) + 3 = 2×(-n + 1) + 1

c. an = 7n - 6

d. an = 7×(-n) + 8 = 7×(-n + 1) + 1

so, please pick the right expression from this list, and then use the answer option with the same letter.

help pls!!

The functionſ is defined by f(x) = x(x + 3). If f(a) = 40 and a > 0, what is the value of a ?
A) 3
B) 5
C) 7
D) 8

Answers

Answer:

B) 5

Step-by-step explanation:

We are given the function:

[tex]f(x)=x(x+3)[/tex]

We are given that f(a) = 40 and a > 0 and we want to determine the value of a.

Substitute:

[tex]f(a)=40=a(a+3)[/tex]

Distribute:

[tex]a^2+3a=40[/tex]

Subtract 40 from both sides:

[tex]a^2+3a-40=0[/tex]

We can factor using 8 and -5. Hence:

[tex](a+8)(a-5)=0[/tex]

By the Zero Product Property:

[tex]a+8=0\text{ or } a-5=0[/tex]

Solve for each case:

[tex]\displaystyle a=-8\text{ or } a=5[/tex]

Since a > 0, we can eliminate the first solution. Hence:

[tex]a=5[/tex]

Our answer is B.

Express each of the following decimal number in p/q form
3.127bar​

Answers

Answer:

3124/999

Step-by-step explanation:

x = 3. 127 127 ....... -------------(I)

127 -----> 3 digits are repeating. so multiply both sides by 1000

(I)*1000             1000x = 3127.127127......

                                 x =       3.127127......        [Now subtract}

                           999x = 3124

x = 3124/999

Triangle Angle-Sum Theorem

Answers

Answer:

answer is C , 123

Step-by-step explanation:

angle 1 = sum of the interior opposite Angles

1 = 60 + 63

1 = 123

Find the intersection point between the lines of equations:

2x-y+6=0 and 2x+3y-6=0 ​

Answers

Step-by-step explanation:

The two equation will intersect each other at the point which will be the solution of the given two equations , and the given equations are ,

[tex]\implies 2x -y +6=0\\\\\implies 2x + 3y -6=0[/tex]

On subtracting the given equations we have,

[tex]\implies -y - 3y +6 -(-6) = 0 \\\\\implies -4y = -12 \\\\\implies y = -12/-4\\\\\implies y = 3 [/tex]

Put this value in any equation , we have ,

[tex]\implies 2x -3 +6 =0\\\\\implies 2x = -3 \\\\\implies x =\dfrac{-3}{2} \\\\\implies x =-1.5 [/tex]

Hence the lines will Intersect at ,

[tex]\implies\underline{\underline{ Point=(-1.5, 3)}}[/tex]

for the first one x = 1/2 y-3" and y = 2 x + 6 and for the other one is x =  − 3 /2  y+ 3  and y= − 2 /3  x + 2

how i did this Step 1: Add -3y to both sides.

Step 2: Add 6 to both sides.

Step 3: Divide both sides by 2.

please say me the answer fast with step by step process​

Answers

Yes I’m sorry but I’m sorry I’m texting back to my sister in

Can someone help me with this math homework please!

Answers

Answer:

the correct options are 1, 2, 5, and 6th

1. and 2.

all functions have a dependant variable y and independent variable x, and the set of dependant variables is called its range whereas the set of independent variables is called its domain.

5. and 6.

In a horizontal line 1 input associates itself with exactly one ouput, even tho the output is constant (same) for all values of x, and that's the property of a function

Answer:

all functions have dependent variables

all functions have an independent variable

a horizontal line Is an example of a functional relationship

The 20th term of an arithmetic progression 1,5,9,13......​

Answers

Answer:

a₂₀ = 77

Step-by-step explanation:

There is a common difference between consecutive terms , that is

5 - 1 = 9 - 5 = 13 - 9 = 4

This indicates the sequence is arithmetic with nth term

[tex]a_{n}[/tex] = a₁ + (n - 1)d

where a₁ is the first term and d the common difference

Here a₁ = 1 and d = 4 , then

a₂₀ = 1 + (19 × 4) = 1 + 76 = 77

Suppose that X1 and X2 are independent random variables each with a mean μ and a variance σ^2. Compute the mean and variance of Y = 3X1 + X2.

Answers

Mean:

E[Y] = E[3X₁ + X₂]

E[Y] = 3 E[X₁] + E[X₂]

E[Y] = 3µ + µ

E[Y] = 4µ

Variance:

Var[Y] = Var[3X₁ + X₂]

Var[Y] = 3² Var[X₁] + 2 Covar[X₁, X₂] + 1² Var[X₂]

(the covariance is 0 since X₁ and X₂ are independent)

Var[Y] = 9 Var[X₁] + Var[X₂]

Var[Y] = 9σ² + σ²

Var[Y] = 10σ²

Which operation will solve the following word problem? Jeff earns $14.00 per hour, Tom earns half as much as Jeff. How much does Tom earn per hour?


Multiplication


Subtraction


Addition


Division

Answers

Answer:

Tom earns $7.00 per hour

Its Subtraction
………………………….


















.

Calculate the average rate of change of a function over a specified interval. Which expression can be used to determine the average rate of change in f(x) over the interval 2, 9?

Answers

Given:

The interval is [2,9].

To find:

The average rate of change in f(x) over the interval [2,9].

Solution:

The average rate of change in f(x) over the interval [a,b] is defined as:

[tex]m=\dfrac{f(b)-f(a)}{b-a}[/tex]

In the interval [2,9], the value of a is 2 and the value of b is 9.

Using the above formula, the average rate of change in f(x) over the interval [2,9] is:

[tex]m=\dfrac{f(9)-f(2)}{9-2}[/tex]

[tex]m=\dfrac{f(9)-f(2)}{7}[/tex]

Therefore, the required expression for the average rate of change in f(x) over the interval [2,9] is [tex]\dfrac{f(9)-f(2)}{9-2}[/tex], it is also written is [tex]\dfrac{f(9)-f(2)}{7}[/tex].

Answer:

D

Step-by-step explanation:

right on edge

Find the missing angle in the image below. Do not include spaces in your answers ** Can somebody help me fr everybody keep giving me the wrong answer

Answers

Answer:

measure of angle V + measure angle W = VUF

Answer:

∠ VUF = 94°

Step-by-step explanation:

The exterior angle of a triangle is equal to the sum of the 2 opposite interior angles

∠ VUF is an exterior angle of the triangle, then

∠ VUF = 71° + 23° = 94°

write a equation for y=|x| if the graph is translated right by 3 units and down by 1 unit

Answers

Answer:

y=|x -3| -1

Step-by-step explanation:

A cylinder has a volume of 245x cubic units and a helght of 5 units. The diameter of the cylinder is
7 units
14 units
49 units

Answers

Answer:

Diameter = 14 units

Step-by-step explanation:

Volume ofa cylinder = πr²h

Volume of the cylinder = 245π cubic units

Height = 5 units

Volume of a cylinder = πr²h

245π = π × r² × 5

245π = 5r²π

Divide both sides by π

245π / π = 5r²π / π

245 = 5r²

r² = 245/5

= 49

r² = 49

r = √49

r = 7 units

Diameter = 2 × radius

= 2 × 7 units

= 14 units

Diameter = 14 units

FOR EASY BRAINLIEST ANSWER QUESTION BELOW!

1. Solve each word problem .twice a number added three times the sum of the number and 2 is more than 17. Find the numbers that satisfy condition

Answers

Answer:

56

Step-by-step explanation:

What percentage of temperatures are below 55°? A. 50% B. 25% C. 75% D. 20%

Answers

Step-by-step explanation:

C

im not sure of my answer. hope to help

I think d because others one

Dog breeds 2238 pages in seven hours he reads the same number of pages each hour how many pages did he read in one hour

Answers

Answer:

319 pages and a half

Step-by-step explanation:

Divide 2238 by seven. The answer is about 319.71.

Which fraction is the largest? 7/9 3/4 1/2 2/3

Answers

Rewrite the fractions as decimals by dividing:

7/8 = 0.7777

3/4 = 0.75

1/2 = 0.5

2/3 = 0.666

0.777 is the largest number so 7/9 is the largest fraction.

Answer: 7/9

Answer:

2/3

Step-by-step explanation:

L.C.M:36

7/9:28/36

3/4:27/36

1/2:1836

2/3:36/36

the first day she walked 27 kilometers. each day since she walked 2/3 of what she walked the day before. what is the total distance cecelia has traveled be the end of the 5th day?

Answers

Answer: 70

Step-by-step explanation:

We are required to calculate the total distance Cecilia travelled in 5 days

The total distance Cecilia travelled for 5 days is 99 kilometers

Day 1 = 27 kilometers

Day 2 to day 5 = 2/3 of 27

= 2/3 × 27

= 2 × 9

= 18 kilometers each day

Total distance = day 1 + day 2 + day 3 + day 4 + day 5

= (27 + 18 + 18 + 18 + 18) kilometers

= 99 kilometers

Therefore, the total distance Cecilia travelled for 5 days = 99 kilometers

Read more:

https://brainly.com/question/17207658

What degree of rotation about the origin will cause the triangle below to map
onto itself?
A.90
B.270
C.360
D.180

Answers

Sorry I don’t see any images or diagrams I’m not sure
360° Would cause the origin to map back onto itself, C. Hope this helps!

Write an equation that represents the line.
Use exact numbers.

Answers

the line passes point (0, -3) and (1, -5)

the slope = (-5+3) /(1-0) = -2/1 = -2

the y-intercept = -3

so, the equation : y = -2x -3

What number is increased by 40% become 28?
a.5 b. 10 c. 15 d. 20​

Answers

Answer:

d. 20

Step-by-step explanation:

28 ÷ (100%+40%)

= 28 ÷ 140%

= 28 ÷ 1.4 = 20

Answer:

option d

Step-by-step explanation:

Let the number be x

40% of x = 0.40x

Increased by 40% = 28

That is ,

           x + 0.40x = 28

               1.40x = 28

                   [tex]x = \frac{28}{1.40}\\[/tex]

                  [tex]x = \frac{28 \times 100}{1.40 \times 100 } = \frac{28 \times 100}{140} =\frac{4 \times 100}{20} = 4 \times 5 = 20[/tex]

Therefore, the number is 20

use the substitution method to solve the system of equations

3x+2y=13

y=x-1

Answers

Answer:

[tex]x=3\\y=2[/tex]

Step-by-step explanation:

[tex]3x+2y=13\\y=x-1[/tex]

Solve for [tex]y=x-1[/tex] for y:

[tex]y=x-1[/tex]

Substitute [tex]x-1[/tex] for y in [tex]3x+2y=13[/tex]:

[tex]3x+2y=13[/tex]

[tex]3x+2(x-1)=13\\[/tex]

[tex]5x-2=13[/tex]

[tex]5x=13+2[/tex]

[tex]5x=15[/tex]

[tex]x=15/5[/tex]

[tex]x=3[/tex]

Substitute 3 for x in [tex]y=x-1[/tex]:

[tex]y=x-1[/tex]

[tex]y=(3)-1[/tex]

[tex]y=2[/tex]

[tex]x=3[/tex] and [tex]y=2[/tex]

hope this helps...

Find the measure of ∠AED for m∠BEC = 118

Answers

Answer: 118°

Step-by-step explanation:

Given EH = 12 and EG = 20 , find FJ.

Answers

Answer:

10

Step-by-step explanation:

If you look at it 20 is from EG so if FJ is half of EG then FJ is half of 20

the regular price of base ball cleats is 80 dollars if the cleats are on sale for 45% off, then what is the value of the discount in dollars?

Answers

Answer:

$36

Step-by-step explanation:

If the cleats were on sale for 45% off, that means they were 55% "on." Therefore, you have to multiply 80 by 0.55* to get how much they cost.

80 x 0.55 is 44.

To get the value of the discount, you subtract 44 from 80, which is 36.

The cleats sold for $44, and there was a $36 discount.

*you multiply by 0.55 because it is 55 percent. Percent means "per every hundred." That's why you have to divide 55 by 100 to get 0.55.

An alternate way to solve this is to multiply 80 by 55 and then divide the result by 100, but you'll still get 44.

Other Questions
What is unit? Write down the units of mass, temperature and power Subtract [tex] - 3[/tex][tex] - {2y}^{3} [/tex][tex] - y[/tex][tex] - {5y}^{2} [/tex] from[tex] - {2y}^{3} [/tex][tex] + 4[/tex] Which step shows the result of applying the subtraction property of equality?(12x+8)+4-3 HELP!!! I'LL GIVE BRAINLIEST FOR ALL THE CORRECT ANSWERS IN THIS!! RELIGIOUS EDUCATION Help ASAP only right answers only no spam dont answer if you dont know B 2 -3 -2 -1 Use the Pythagorean theorem to find the distance between points A and B on each graph. round answers to the nearest tenth. Z varies directly as Square x and inversely as y. If z = 187 when x = 64 and y = 6, find z if and 9. (Round off your answer to the nearest hundredth.) A stockbroker has kept a daily record of the value of a particular stock over the years and finds that prices of the stock form a normal distribution with a mean of $8.52 with a standard deviation of $2.38. The stock price beyond which 0.05 of the distribution falls is _________. One of the most important aspects of visual aids is: Group of answer choices They are self-explanatory They complement a presentation or speech They should not be used in a professional setting They replace spoken words in a presentation In the 2008 global financial crisis, many investors considered the US economy a safe place to move their assets What is the predicted impact of this inflow of financial capital to the US, which is a large, open economy, on the US interest rate and the US exchange rate, holding other factors constant Illustrate your answer graphically and explain in words. Change "She loves Burger" to passive Suppose 58% of the population has a retirement account. If a random sample of size 570 is selected, what is the probability that the proportion of persons with a retirement account will be less than 57% Does anyone know the answer? with steps please A student uses a clinometer to measure the angle of elevation of a sign that marks the point on a tower that is 45 m above the ground. The angle of elevation is 32 and the student holds the clinometer 1.3 m above the ground. He then measures the angle of elevation of the top of the tower as 47. Sketch and label a diagram to represent the information in the problem. Determine the height of the tower to the nearest tenth of a metre 45 people were surveyed. 33 people like hamburgers, 18 people like hamburgers and hot dogs. How many people like hot dogs? Graph g(x)=-8|x |+1. please help to answer thus question asap!! please Can anyone help with this She decides that ordering that many cars would not be economically feasible at this time and asks her sales manager to randomly choose one of the models for the sales lot. What is the probability that he chooses the 4-door, special edition model, with four-wheel drive?The probability that he chooses the 4-door, special edition, four-wheel drive model is (44)=P(4S4)= Brian is attempting to determine whether it makes sense to think that the human need to belong and to establish relationships has an evolutionary basis. Based on the principles of natural selection, Brian knows he could support this idea if there was evidence that